2019 AMC 10A Problems/Problem 3
2019 AMC 10A Problems/Problem 3
2019 AMC 10A Problems/Problem 3
Problem 1
Solution
Solution
The last three digits of for all are , because there are at least
three s and three s in its prime factorization. Because , the
answer is .
Solution 1
Let be the age of Ana and be the age of Bonita. Then,
and
dividing, Moreover,
The answer is
Problem
A box contains red balls, green balls, yellow balls, blue
balls, white balls, and black balls. What is the minimum number of balls
that must be drawn from the box without replacement to guarantee that at
least balls of a single color will be drawn
Solution
By choosing the maximum number of balls while getting of each color,
we could have chosen red balls, green balls, yellow
balls, blue balls, white balls, and black balls, for a total of balls.
Picking one more ball guarantees that we will get balls of a color -- either
Problem
What is the greatest number of consecutive integers whose sum is
Solution 1
We might at first think that the answer would be ,
as low as possible while still being positive. The average can be if the
§ a square
§ a rectangle that is not a square
§ a rhombus that is not a square
§ a parallelogram that is not a rectangle or a rhombus
§ an isosceles trapezoid that is not a parallelogram
Solution 1
This question is simply asking how many of the listed quadrilaterals are cyclic
(since the point equidistant from all four vertices would be the center of the
circumscribed circle). A square, a rectangle, and an isosceles trapezoid (that isn't
a parallelogram) are all cyclic, and the other two are not. Thus, the answer
is .
Solution 2
We can use a process of elimination. Going down the list, we can see a square
obviously works. A rectangle that is not a square works as well. Both rhombi and
parallelograms don't have a point that is equidistant, but isosceles trapezoids do
is .
Problem
Two lines with slopes and intersect at . What is the area of the
Solution 1
Let's first work out the slope-intercept form of all three
so ,
while implies so .
which are and . Using the distance formula and then the
Pythagorean Theorem, we see that we have an isosceles triangle with
Solution 2
Like in Solution 1, we determine the coordinates of the three vertices of the
triangle. Now, using the Shoelace Theorem, we can directly find that the area
is .
Solution 3
Like in the other solutions, solve the systems of equations to see that the
Solution 4
Like in the other solutions, we find, either using algebra or simply by drawing
the lines on squared paper, that the three points of intersection
triangle's area is
Problem
The figure below shows line with a regular, infinite, recurring pattern of
squares and line segments.
How many of the following four kinds of rigid motion transformations of the
plane in which this figure is drawn, other than the identity transformation, will
transform this figure into itself?
Solution
Statement is true. A rotation about the point half way between an
up-facing square and a down-facing square will yield the same figure.
Statement is also true. A translation to the left or right will place the image
onto itself when the figures above and below the line realign (the figure goes
on infinitely in both directions).
Statement is false. A reflection across line will change the up-facing
squares to down-facing squares and vice versa.
Finally, statement is also false because it will cause the diagonal lines
extending from the squares to switch direction. Thus,
Solution 2
As in Solution 1, we deduce that must be prime. If we can't immediately
recall what the greatest three-digit prime is, we can instead use this result to
eliminate answer choices as possible values of . Choices , , and don't
work because is even, and choice does not work since is
Solution 1
The number of tiles the bug visits is equal to plus the number of times it
crosses a horizontal or vertical line. As it must cross horizontal lines
and vertical lines, it must be that the bug visits a total
of squares.
Solution 2 (drawing)
We draw a diagram (optionally with grid paper and/or a ruler), then simply count
the number of tiles the path crosses. To make this slightly easier, we can divide
the full grid into sections, and just draw one of these feet by feet
sections.
How many positive integer divisors of are perfect squares or perfect cubes
(or both)?
Solution 1
and ), we get .
Solution 2
Observe that . Now divide into cases:
Case 1: The factor is . Then we can have , , , , , or .
Case 2: The factor is . This is the same as Case 1.
Case 3: The factor is some combination of s and s.
This would be easy if we could just have any combination, as that would simply
give . However, we must pair the numbers that generate squares with the
numbers that generate squares and the same for cubes. In simpler terms, let's
organize our values for .
is a "square" because it would give a factor of this number that is a
perfect square. More generally, it is even.
is a "cube" because it would give a factor of this number that is a perfect
cube. More generally, it is a multiple of .
is a "square".
is interesting, since it's both a "square" and a "cube". Don't count this as
either because this would double-count, so we will count this in another case.
is a "square"
is a "cube".
Now let's consider subcases:
Subcase 1: The squares are with each other.
Since we have square terms, and they would pair with other square terms,
we get possibilities.
Subcase 2: The cubes are with each other.
Since we have cube terms, and they would pair with other cube terms, we
get possibilities.
Subcase 3: A number pairs with .
Since any number can pair with (as it gives both a square and a cube),
there would be possibilities. Remember however that there can be two different
bases ( and ), and they would produce different results. Thus, there are in
fact possibilities.
Problem
Melanie computes the mean , the median , and the modes of
the values that are the dates in the months of . Thus her data
consist of , ,..., , , , and .
Let be the median of the modes. Which of the following statements is true?
Solution 1
First of all, obviously has to be smaller than , since when
calculating , we must take into account the s, s, and s. So we
can eliminate choices and . Since there are total entries, the
median, , must be the one, at which point we note
that is , so has to be the median (because is
answer is .
diameter , and let and be the other intersection points of the circle
Solution 1
Drawing it out, we see and are right angles, as they are
inscribed in a semicircle. Using the fact that it is an isosceles triangle, we
find . We can
find and by the triangle angle sum
on and .
find
Solution 2
Alternatively, we could have used similar triangles. We start similarly to Solution
1.
Drawing it out, we see and are right angles, as they are
inscribed in a semicircle.
Therefore,
So, by AA Similarity,
since and .
Thus, we know
Finally, we
deduce
Solution 3 (outside angles)
Through the property of angles formed by intersecting chords, we find
that
that
Thus
Solution 4
Notice that if , then and must be .
Using cyclic quadrilateral properties (or the properties of a subtended arc), we
can find that .
Thus , and
so , which is .
Note: As in many elementary geometry problems, if you can't see how to solve it,
you could simply draw an accurate diagram and measure the angle using a
protractor as .
Solution
It is possible to obtain , , , , , and points of intersection, as
demonstrated in the following
figures:
All possibilities have been exhausted, and thus we can conclude that two
intersections is impossible. Our answer is given by the
sum .
Solution 1
Since all four terms on the left are positive integers, from , we know
that both has to be a perfect square and has to be a power of ten. The
same applies to for the same reason. Setting and to and ,
seen: , , ,
Solution 2
Therefore, .
Problem
The figure below shows circles of radius within a larger circle. All the
intersections occur at points of tangency. What is the area of the region,
shaded in the figure, inside the larger circle but outside all the circles of
radius
Solution 1
In the diagram above, notice that triangle and triangle are
congruent and equilateral with side length . We can see the radius of the
larger circle is two times the altitude of plus (the distance from
point to the edge of the circle). Using triangles,
is .
is .
Solution 2
We can form an equilateral triangle with side length from the centers of
three of the unit circles tangent to the outer circle. The radius of the outer
circle is the circumradius of the triangle plus . By
Solution 3
Like in Solution 2, we can form an equilateral triangle with side length from
the centers of three of the unit circles tangent to the outer circle. We can find
.
The total area of the smaller circles is , so the shaded area
is .
Solution 4
In the diagram above, and ,
is .
Now, subtracting the combined area of the smaller circles
gives .
as
Solution 2
We can divide the problem into three cases, each representing one cube to be
excluded:
Case 1: The red cube is excluded. This gives us the problem of arranging one
red cube, three blue cubes, and four green cubes. The number opossible
arrangements is .
Case 3: The green cube is excluded. This gives us the problem of arranging two
red cubes, three blue cubes, and three green cubes. The number of possible
arrangements is .
Adding up the individual cases from above gives the answer
as .
Solution 3 (guessing)
If you're running out of time, notice that choices , , and are way too
small, and choice would make no sense since it would simply be , as if
there were no restrictions. Thus, by educated guessing and elimination, the
Problem
For some positive integer , the repeating base- representation of the
Solution 1
Solution 2
Let . Therefore, .
so .
is .
Solution 3 (bash)
We can simply plug in all the answer choices as values of , and see which
Solution 4
Notice that .
gives .
of where is a real
number?
Solution 1
Grouping the first and last terms and two middle terms
answer is .
Solution 2
expression becomes
get .
is , so .
Note: We could also have used the result that minimum/maximum point of a
parabola occurs at .
Solution 4
The expression is negative when an odd number of the factors are negative. This
happens when or . Plugging
answer is .
Problem
Solution 1
Solution 2
By the Pigeonhole Principle, there must be at least one row with or more
odd numbers in it. Therefore, that row must contain odd numbers in order
to have an odd sum. The same thing can be done with the columns. Thus we
simply have to choose one row and one column to be filled with odd numbers,
so the number of valid odd/even configurations (without regard to which
particular odd and even numbers are placed where) is . The
is
Problem
A sphere with center has radius . A triangle with sides of
Diagram
3D: Plane
through triangle:
Solution 1
The triangle is placed on the sphere so that its three sides are tangent to the
sphere. The cross-section of the sphere created by the plane of the triangle is
also the incircle of the triangle. To find the inradius,
find .
Solution 2
As in Solution 1, we note that by the Pythagorean Theorem, the height of the
triangle is , and that the three sides of the triangle are tangent to the sphere,
so the circle in the cross-section of the sphere is the incenter of the triangle.
Recall that the inradius is the intersection of the angle bisectors. To find the
inradius of the incircle, we use the Angle Bisector
Theorem.
Using the Pythagorean Theorem, we find that the distance from the center to
the plane is .
Problem
Real numbers between 0 and 1, inclusive, are chosen in the following
manner. A fair coin is flipped. If it lands heads, then it is flipped again and the
chosen number is 0 if the second flip is heads and 1 if the second flip is tails.
On the other hand, if the first coin flip is tails, then the number is chosen
probability that ?
Solution
There are several cases depending on what the first coin flip is when
determining and what the first coin flip is when determining .
The four cases are:
Case 1: is either or , and is either or .
the interval .
Each case has a chance of occurring (as it requires two coin flips).
success in Case 1 is .
is (since the total area of the bounding square, containing all possible
pairs, is ).
Adding up the success rates from each case, we get:
Notice that at the end of the , the last number said is the triangular
number.
Tadd says number in round 1, numbers in round 2, numbers in round 3,
and in general numbers in round . At the end of round , the number
of numbers Tadd has said so far
is , by the arithmetic
series sum formula.
We therefore want the smallest positive integer such
says his number. Through guess and check (or by actually solving the
quadratic inequality), .
Solution 2
Firstly, as in Solution 1, we list how many words Tadd says, Todd says, and
Tucker says in each round.
Tadd:
Todd:
Tucker:
We can find a general formula for the number of numbers each of the kids say
after the th round. For Tadd, we can either use the arithmetic series sum
formula (like in Solution 1) or standard summation results to
get
.
Now, to find the number of rotations Tadd and his siblings go through before
Tadd says his th word, we know the
that for
all . What is ?
Solution 1
Multiplying both sides
by yields
As this is a polynomial identity, and it is true for infinitely many , it must be true
for all (since a polynomial with infinitely many roots must in fact be the constant
polynomial ). This means we can plug in to find
that By Vieta's
Formulas, we know
that
a
is .
Note: This process of substituting in the 'forbidden' values in the original identity
is a standard technique for partial fraction decomposition, as taught in calculus
classes.
Solution 2 (limits)
that
As , notice that the and terms on the right will cancel out and we
becomes . We can
reason similarly to find and . Adding up the reciprocals and using Vieta's
Formulas, we have
that
Problem
For how many integers between and , inclusive, is an
integer? (Recall that .)
Solution
The main insight is that
is potentially not an integer. It can be easily verified that the above expression
is not an integer for as there are more factors of 2 in the denominator
than the numerator. Similarly, it can be verified that the above expression is
not an integer for any prime , as there are more factors of p in the
denominator than the numerator. Thus all 16 values of n make the expression